thank you, Mirko!

This commit is contained in:
Josia Pietsch 2024-02-14 13:35:06 +01:00
parent deaf6a6bb6
commit 11f21af966
Signed by: josia
GPG Key ID: E70B571D66986A2D
7 changed files with 37 additions and 32 deletions

View File

@ -124,9 +124,12 @@ together with the additional axiom:
\begin{fact} \begin{fact}
$\BGC$ is conservative over $\ZFC$, $\BGC$ is conservative over $\ZFC$,
i.e.~for all formulae $\phi$ in the language i.e.~for all formulae $\phi$ in the language
of set theory (only set variables): of set theory (only set variables)
we have that
if $\BGC \vdash \phi$ then $\ZFC \vdash \phi$. if $\BGC \vdash \phi$ then $\ZFC \vdash \phi$.
\end{fact} \end{fact}
We cannot prove this fact at this point, We cannot prove this fact at this point,
as the proof requires forcing. as the proof requires forcing.
The converse is easy however, i.e.~if The converse is easy however, i.e.~if
@ -172,9 +175,9 @@ $\ZFC \vdash \phi$ then $\BGC \vdash \phi$.
\end{lemma} \end{lemma}
\begin{proof} \begin{proof}
Suppose such sequence exists. Suppose such sequence exists.
Then $\{x_n : n < \omega\}$ Then $\{x_n : n < \omega\}$%
(this exists as by definition sequence of the $x_n$ is a function \footnote{This exists as by definition the sequence $(x_n)$ is a function
and this set is the range of that function) $f\colon \omega \to V$ and this set is the image of $f$.}
violates \AxFund. violates \AxFund.
For the other direction let $M \neq \emptyset$ be some set. For the other direction let $M \neq \emptyset$ be some set.

View File

@ -4,12 +4,12 @@
\begin{definition} \begin{definition}
Let $R$ be a binary relation. Let $R$ be a binary relation.
$R$ is called \vocab{well-founded} $R$ is called \vocab{well-founded}
if for all classes $X$, iff for all classes $X$,
there is an $R$-least $y$ such that there is an $R$-least $y$ such that
there is no $z \in X$ with $(z,y) \in R$. there is no $z \in X$ with $(z,y) \in R$.
\end{definition} \end{definition}
\begin{theorem}[Induction (again, but now with classes)] \begin{theorem}[Induction (again, but now for classes)]
Suppose that $R$ is a well-founded relation. Suppose that $R$ is a well-founded relation.
Let $X$ be a class such that for all sets $x$, Let $X$ be a class such that for all sets $x$,
\[ \[
@ -22,7 +22,7 @@
Consider $Y = \{x : x \not\in X\} \neq \emptyset$. Consider $Y = \{x : x \not\in X\} \neq \emptyset$.
By hypothesis, there is some By hypothesis, there is some
$x \in Y$ such that $(y,x) \not\in R$ for all $y \in Y$. $x \in Y$ such that $(y,x) \not\in R$ for all $y \in Y$.
In other words, if $(y,x) \not\in R$, In other words, if $(y,x) \in R$,
then $x \not\in Y$, i.e.~$x \in X$. then $x \not\in Y$, i.e.~$x \in X$.
Thus $\{y: (y,x) \in R\} \subseteq X$. Thus $\{y: (y,x) \in R\} \subseteq X$.
Hence $x \in X \lightning$. Hence $x \in X \lightning$.

View File

@ -52,7 +52,7 @@ We often write $\kappa, \lambda, \ldots$ for cardinals.
Let us now assume that for all $\kappa \in X$ Let us now assume that for all $\kappa \in X$
there is some $\lambda \in X$ there is some $\lambda \in X$
with $\lambda > \kappa$. with $\lambda > \kappa$.
Suppose that $\sup(X)$ is no a cardinal Suppose that $\sup(X)$ is not a cardinal
and write $\mu = |\sup(X)|$. and write $\mu = |\sup(X)|$.
Then $\mu \in \sup(X)$, Then $\mu \in \sup(X)$,
since $\sup(X)$ is an ordinal. since $\sup(X)$ is an ordinal.
@ -96,7 +96,7 @@ so $|a| = \aleph_\beta$ for some $\beta \le |\alpha|$.
\end{notation} \end{notation}
\begin{notation} \begin{notation}
Let $\leftindex^a b \coloneqq \{f : f \text{ is a function}, \dom(f) = \alpha, \ran(f) \subseteq b\}$. Let $\leftindex^a b \coloneqq \{f : f \text{ is a function}, \dom(f) = a, \ran(f) \subseteq b\}$.
\end{notation} \end{notation}
\begin{definition}[Cardinal arithmetic] \begin{definition}[Cardinal arithmetic]

View File

@ -215,15 +215,17 @@ cf.~\yaref{def:inaccessible}.
\end{IEEEeqnarray*} \end{IEEEeqnarray*}
\begin{itemize} \begin{itemize}
\item First case: $\beta \ge \alpha+1$. \item First case: $\beta \ge \alpha+1$.
Then Note that for all $\gamma \le \beta$
we have
\[ \[
\aleph_{a+1}^{\aleph_\beta} \le \aleph_{\beta}^{\aleph_\beta} \aleph_{\gamma}^{\aleph_\beta} \le \aleph_\beta^{\aleph_\beta}
\le \left( 2^{\aleph_{\beta}} \right)^{\aleph_\beta} \le \left( 2^{\aleph_\beta} \right)^{\aleph_\beta}
= 2^{\aleph_{\beta} \cdot \aleph_\beta} = 2^{\aleph_\beta} \le \aleph_{\alpha+1}^{\aleph_\beta}. = 2^{\aleph_\beta \cdot \aleph_\beta} = 2^{\aleph_{\beta}}
\le \aleph_\gamma^{\aleph_\beta}.
\] \]
Also $\aleph_\alpha^{\aleph_{\beta}} = 2^{\aleph_\beta}$ So in this case $\aleph_\alpha^{\aleph_{\beta}} = 2^{\aleph_\beta}$
in this case (by the same argument), and $\aleph_{\alpha+1}^{\aleph_\beta} = 2^{\aleph_{\beta}}$.
so Thus
\[ \[
\aleph_{\alpha+1}^{\aleph_{\beta}} = 2^{\aleph_{\beta}} \aleph_{\alpha+1}^{\aleph_{\beta}} = 2^{\aleph_{\beta}}
= \aleph_{\alpha}^{\aleph_\beta} = \aleph_{\alpha}^{\aleph_\beta} \cdot \aleph_{\alpha+1}. = \aleph_{\alpha}^{\aleph_\beta} = \aleph_{\alpha}^{\aleph_\beta} \cdot \aleph_{\alpha+1}.

View File

@ -4,7 +4,7 @@
There are many well-orders on $\omega$. There are many well-orders on $\omega$.
Let $W$ be the set of all such well-orders. Let $W$ be the set of all such well-orders.
For $R, S \in W$, For $R, S \in W$,
write $R \le S$ if $R$ is isomorphic to write $R \le S$ iff $R$ is isomorphic to
an initial segment of $S$. an initial segment of $S$.
Consider $\faktor{W}{\sim}$, Consider $\faktor{W}{\sim}$,
where $R \sim S :\iff R \le S \land S \le R$. where $R \sim S :\iff R \le S \land S \le R$.
@ -15,7 +15,7 @@
Suppose that $\{R_n : n \in \omega\} \subseteq W$ Suppose that $\{R_n : n \in \omega\} \subseteq W$
is such that $R_{n+1} < R_n$. is such that $R_{n+1} < R_n$.
Then there exist $n_i \in \omega$ Then there exist $n_i \in \omega$
such that $R_i \cong R_0\defon{x <_{R_0} n_i}$ such that $R_i \cong R_0\defon{\{x : x <_{R_0} n_i\}}$
and these form a $<_{R_0}$ strictly decreasing sequence. and these form a $<_{R_0}$ strictly decreasing sequence.
So $(\faktor{W}{\sim})$ So $(\faktor{W}{\sim})$
@ -116,13 +116,13 @@
then $\cf(2^{\kappa}) = 2^{\kappa} > \kappa$, then $\cf(2^{\kappa}) = 2^{\kappa} > \kappa$,
since successor cardinals are regular. since successor cardinals are regular.
Suppose $\cf(2^{k}) \le \kappa$ is a limit cardinal. Suppose $\cf(2^{\kappa}) \le \kappa$ is a limit cardinal.
Then there is some cofinal $f\colon \kappa\to 2^{\kappa}$. Then there is some cofinal $f\colon \kappa\to 2^{\kappa}$.
Write $\kappa_i = f(i)$ Write $\kappa_i = f(i)$
(replacing $f(i)$ by $|f(i)|^{+}$ we may assume (replacing $f(i)$ by $|f(i)|^{+}$ we may assume
that every $\kappa_i$ is a cardinal). that every $\kappa_i$ is a cardinal).
For $i \in \kappa$, write $\lambda_i = 2^{k}$. For $i \in \kappa$, write $\lambda_i = 2^{\kappa}$.
By \yaref{thm:koenig}, By \yaref{thm:koenig},
\[ \[
\sup \{\kappa_i : i < \kappa\} \le \sum_{i \in \kappa} \kappa_i < \prod_{i \in \kappa} \lambda_i = \left( 2^{\kappa} \right)^{\kappa} = 2^{\kappa \cdot \kappa} = 2^{\kappa} \sup \{\kappa_i : i < \kappa\} \le \sum_{i \in \kappa} \kappa_i < \prod_{i \in \kappa} \lambda_i = \left( 2^{\kappa} \right)^{\kappa} = 2^{\kappa \cdot \kappa} = 2^{\kappa}
@ -171,14 +171,14 @@ Relevant concepts to prove this theorem:
\item We say that $A \subseteq \alpha$ \item We say that $A \subseteq \alpha$
is \vocab{unbounded} (in $\alpha$), is \vocab{unbounded} (in $\alpha$),
iff for all $\beta < \alpha$, iff for all $\beta < \alpha$,
there is some $\gamma \in \alpha$ there is some $\gamma \in A$
such that $\beta < \gamma$. such that $\beta < \gamma$.
\item We say that $A \subseteq \alpha$ \item We say that $A \subseteq \alpha$
is \vocab{closed}, is \vocab{closed},
iff it is closed with respect to the order topology on $\alpha$, iff it is closed with respect to the order topology on $\alpha$,
i.e.~for all $\beta < \alpha$, i.e.~for all $\beta < \alpha$,
\[ \[
\sup(A \cap \beta) \in A. \sup(A \cap \beta) \in A \cup \{0\} .
\] \]
\item $A$ is \vocab{club} (\emph{cl}osed \emph{un}bounded) \item $A$ is \vocab{club} (\emph{cl}osed \emph{un}bounded)
iff it is closed and unbounded. iff it is closed and unbounded.

View File

@ -77,7 +77,7 @@ Recall the following:
of arbitrary sizes. of arbitrary sizes.
How do we do this? How do we do this?
Let $\phi$ be a formula. Let $\phi$ be a formula.
A \vocab{Skolem-function} A \vocab{Skolem-function}
over $V_\theta$ for $\phi$ over $V_\theta$ for $\phi$
is a function is a function
\[ \[
@ -136,7 +136,7 @@ Let's do a second proof of \yaref{thm:fodor}.
\] \]
Note that $|X_{\xi}| = |X_{\xi + 1}|$ Note that $|X_{\xi}| = |X_{\xi + 1}|$
but the size is increased at limits. but the size may increase at limits.
It is easy to see inductively that $|X_{\xi}| < \kappa$ It is easy to see inductively that $|X_{\xi}| < \kappa$
for every $\xi < \kappa$, for every $\xi < \kappa$,
while $X_\xi \subsetneq X_{\xi'}$ while $X_\xi \subsetneq X_{\xi'}$
@ -158,7 +158,7 @@ Let's do a second proof of \yaref{thm:fodor}.
Let $\zeta < \kappa$. Let $\zeta < \kappa$.
Let us define a strictly increasing sequence Let us define a strictly increasing sequence
$ \langle \xi_n : n < \omega \rangle$ $ \langle \xi_n : n < \omega \rangle$
a follows. as follows.
Set $\xi_0 \coloneqq \zeta$. Set $\xi_0 \coloneqq \zeta$.
Suppose $\xi_n$ has been chosen. Suppose $\xi_n$ has been chosen.
Look at $X_{\xi_n} \cap \kappa$. Look at $X_{\xi_n} \cap \kappa$.

View File

@ -41,7 +41,7 @@ where $f(\alpha) = \gamma$.
\gist{% \gist{%
Recall that $F \subseteq \cP(\kappa)$ is a filter if Recall that $F \subseteq \cP(\kappa)$ is a filter if
$X,Y \in F \implies X \cap Y \in F$, $X,Y \in F \implies X \cap Y \in F$,
$X \in X, X \subseteq Y \subseteq \kappa \implies Y \in F$ $X \in F, X \subseteq Y \subseteq \kappa \implies Y \in F$
and $\emptyset \not\in F, \kappa \in F$. and $\emptyset \not\in F, \kappa \in F$.
\todo{Move this to the definition of filter?} \todo{Move this to the definition of filter?}
}{} }{}
@ -86,7 +86,7 @@ one cofinality.
If $S \subseteq \kappa$ If $S \subseteq \kappa$
is stationary, is stationary,
there is a sequence $\langle S_i : i < \kappa \rangle$ there is a sequence $\langle S_i : i < \kappa \rangle$
of pairwise disjoint stationary sets of $\kappa$ of pairwise disjoint stationary subsets of $\kappa$
such that $S = \bigcup S_i$. such that $S = \bigcup S_i$.
\end{theorem} \end{theorem}
\begin{corollary} \begin{corollary}
@ -109,12 +109,12 @@ one cofinality.
% TODO: Look at this again and think about it. % TODO: Look at this again and think about it.
% TODO TODO TODO % TODO TODO TODO
We will only proof this for $\aleph_1$. We will only prove this for $\aleph_1$.
Fix $S \subseteq \aleph_1$ stationary. Fix $S \subseteq \aleph_1$ stationary.
For each $0 < \alpha < \omega_1$, For each $0 < \alpha < \omega_1$,
either $\alpha$ is a successor ordinal either $\alpha$ is a successor ordinal
or $\alpha$ is a limit ordinal and $\cf(\alpha) = \omega_1$. or $\alpha$ is a limit ordinal and $\cf(\alpha) = \omega$.
Let $S^\ast \coloneqq \{\alpha \in S \setminus \{0\} : \alpha \text{ is a limit ordinal}\} $. Let $S^\ast \coloneqq \{\alpha \in S \setminus \{0\} : \alpha \text{ is a limit ordinal}\} $.
$S^\ast$ is still stationary: $S^\ast$ is still stationary:
@ -163,7 +163,7 @@ one cofinality.
Let $C' \coloneqq C \setminus (\delta^\ast + 1)$. Let $C' \coloneqq C \setminus (\delta^\ast + 1)$.
$C'$ is still club. $C'$ is still club.
As $\delta^\ast$ is stationary, As $S^\ast$ is stationary,
we may pick some $\alpha \in S^\ast \cap C'$. we may pick some $\alpha \in S^\ast \cap C'$.
But then $\gamma_n^{\alpha} > \delta^\ast$ But then $\gamma_n^{\alpha} > \delta^\ast$
for $n$ large enough for $n$ large enough
@ -285,9 +285,9 @@ However we can say something about singular cardinals:
Recall that $\CH$ says that $2^{\aleph_0} = \aleph_1$. Recall that $\CH$ says that $2^{\aleph_0} = \aleph_1$.
So $\GCH \implies \CH$. So $\GCH \implies \CH$.
\yalabel{thm:silver} says that if $\GCH$ is true below $\kappa$, \yaref{thm:silver} says that if $\GCH$ is true below $\kappa$,
then it is true at $\kappa$. then it is true at $\kappa$.
The proof of \yalabel{thm:silver} is quite elementary, The proof of \yaref{thm:silver} is quite elementary,
so we will do it now, but the statement can only be fully appreciated later. so we will do it now, but the statement can only be fully appreciated later.
}{} }{}